Sample Math Test Questions Flashcards

1
Q

If $100 is deposited today in an account paying 9 percent compound annual interest, how much will be in the account at the end of 4 years?

(A) $70.84

(B) $141.16

(C) $323.97

(D) $457.31

A

The answer is (B).

HP-10BII keystrokes: , C ALL; 100, +/–, PV; 9, I/YR; 4, N; FV. Answer displayed: 141.16. (A), (C), and (D) are incorrect calculations.

How well did you know this?
1
Not at all
2
3
4
5
Perfectly
2
Q

If Bob deposits $8,000 today in an account that pays 3 percent compound annual interest, how long will it take for the account to reach $10,000?

(A) 3.77 years

(B) 4.58 years

(C) 6.12 years

(D) 7.55 years

A

The answer is (D).

HP-10BII keystrokes: , C ALL; 8000, +/–, PV; 10000, FV; 3, I/YR;N. Answer displayed: 7.55. (A), (B), and (C) are incorrect calculations.

How well did you know this?
1
Not at all
2
3
4
5
Perfectly
3
Q

Assume that payments of $1,000 are to be received at the end of each of the next 5 years. On the basis of a 9 percent compound annual interest assumption, what is the series of payments worth today?

(A) $649.93

(B) $1,538.58

(C) $3,889.65

(D) $5,984.71

A

The answer is (C).

HP-10BII keystrokes: , C ALL; , BEG/END (if BEGIN displayed); 1000, PMT; 9, I/YR; 5, N; PV. Answer displayed: –3,889.65. (A), (B), and (D) are incorrect calculations

How well did you know this?
1
Not at all
2
3
4
5
Perfectly
4
Q

How much money do you need to deposit today in a savings account earning 9 percent compound annual interest if your goal is to accumulate $10,000 at the end of 4 years?

(A) $3,086.71

(B) $7,084.25

(C) $7,721.83

(D) $14,115.82

A

The answer is (B).

HP-10BII keystrokes: , C ALL; 10000, FV; 9, I/YR; 4, N; PV. Answer displayed: –7,084.25.

(A), (C), and (D) are incorrect calculations

How well did you know this?
1
Not at all
2
3
4
5
Perfectly
5
Q

Jim needs to accumulate $25,000 in a savings account over the next 5 years. He can make five annual deposits of $4,000 starting today. What compound annual rate of return must the account earn in order for Jim to meet his goal?

(A) 5.00%

(B) 6.12%

(C) 6.67%

(D) 7.53%

A

The answer is (D).

HP-10BII keystrokes: , C ALL; , BEG/END (if BEGIN not displayed);25000, FV; 4000, +/–, PMT; 5, N; I/YR. Answer displayed: 7.53.

How well did you know this?
1
Not at all
2
3
4
5
Perfectly
6
Q

Bill expects to receive $10,000 each year for 4 years beginning one year from today. If Bill can earn 6 percent compound annual interest on these funds, what is the present value of this series of payments?

(A) $34,651.06

(B) $36,298.95

(C) $49,173.24

(D) $52,421.37

A

The answer is (A).

HP-10BII keystrokes: , C ALL; , BEG/END (if BEGIN displayed); 10000, PMT; 6, I/YR; 4, N; PV. Answer displayed: –34,651.06.

(B), (C), and (D) are incorrect calculations.

How well did you know this?
1
Not at all
2
3
4
5
Perfectly
7
Q

If $1,000 is deposited into a savings account at the beginning of each of the next 4 years starting today, how much money will be in the account 4 years from now if no withdrawals are made and the account earns 9 percent compound annual interest?

(A) $3,239.72

(B) $3,531.29

(C) $4,573.13

(D) $4,984.71

A

The answer is (D).

HP-10BII keystrokes: , C ALL; , BEG/END (if BEGIN not displayed); 1000, +/–, PMT; 9, I/YR; 4, N; FV. Answer displayed: 4,984.71.

(A), (B), and (C) are incorrect calculations.

How well did you know this?
1
Not at all
2
3
4
5
Perfectly
8
Q

What is the yield to maturity of a $1,000 face amount bond that currently sells for $875, pays $60 of interest at the end of each year, and matures in 10 years?

(A) 7.85%

(B) 8.75%

(C) 9.35%

(D) 10.00%

A

The answer is (A).

HP-10BII keystrokes: , C ALL; , BEG/END (if BEGIN displayed); 1000, FV; 875, +/–, PV; 60, PMT; 10, N; I/YR. Answer displayed: 7.85.

(B), (C), and (D) are incorrect calculations.

How well did you know this?
1
Not at all
2
3
4
5
Perfectly
9
Q

The liquidity ratio is

(A) Total debt payments divided by gross income

(B) Net worth divided by total assets

(C) Liquid assets divided by total current debts

(D) Net cash flow (after tax) plus savings and investments divided by annual after-tax income

A

The answer is (C).

(A) is incorrect because it is the debt service ratio. (B) is incorrect because it is the solvency ratio. (D) is incorrect because it is the savings ratio.

How well did you know this?
1
Not at all
2
3
4
5
Perfectly
10
Q

Bob wants to accumulate $100,000 in his retirement fund over the next 5 years. He plans to make five equal annual payments into the fund starting today. If the fund can earn 8 percent compound annual interest, how large must each annual payment be?

(A) $10,472.18

(B) $15,783.01

(C) $17,045.65

(D) $68,058.32

A

The answer is (B).

HP-10BII keystrokes: , C ALL; , BEG/END (if BEGIN not displayed);100000, FV; 8, I/YR; 5, N; PMT. Answer displayed: –15,783.01.

(A), (C), and (D) are incorrect calculations.

How well did you know this?
1
Not at all
2
3
4
5
Perfectly
11
Q

Jill plans to make 7 annual payments of $1,000 each to a savings account. Her plan calls for the first payment to be made one year from today. How much money will be in Jill’s account at the end of 7 years if the account earns 4 percent compound annual interest?

(A) $4,246.46

(B) $4,439.94

(C) $7,898.29

(D) $8,654.02

A

The answer is (C).

HP-10BII keystrokes: , C ALL; , BEG/END (if BEGIN displayed); 1000,+/–, PMT; 4, I/YR; 7, N; FV. Answer displayed: 7,898.29.

(A), (B), and (D) are incorrect calculations.

How well did you know this?
1
Not at all
2
3
4
5
Perfectly
12
Q

Which of the following is the effective annual interest rate for a 5 percent nominal rate that is compounded daily?

(A) 5.0945%

(B) 5.1162%

(C) 5.1246%

(D) 5.1267%

A

The answer is (D). HP-10BII keystrokes: , C ALL; , DISP, 4; 5, , NOM%; 365, , P/YR; , EFF%. Answer displayed: 5.1267. Then reset for one payment period per year: 1, , P/YR, C.

(A), (B), and (C) are incorrect calculations.

How well did you know this?
1
Not at all
2
3
4
5
Perfectly
13
Q

Using a discount rate of 6 percent, what is the net present value of an investment with the following cash flow structure? Timing of Cash Flow Amount of Cash Flow Beginning of year 1 $2,000 cash outflow End of year 1 3,000 cash inflow End of year 2 1,000 cash outflow End of years 3 and 4 0 cash flow End of year 5 3,000 cash inflow End of year 6 3,000 cash inflow

(A) $4,296.85

(B) $6,076.84

(C) $8,296.85

(D) $10,076.84

A

The answer is (A).

HP-10BII keystrokes: C ALL; 2000, +/–, CFj; 3000, CFj; 1000, +/–, CFj; 0, CFj,2, , Nj; 3000, CFj, 2, , Nj; 6, I/YR; , NPV. Answer displayed: 4,296.85.

(B), (C), and (D) are incorrect calculations.

How well did you know this?
1
Not at all
2
3
4
5
Perfectly
14
Q

What is the present value (rounded to the nearest dollar) of $12,000 due in 6 years discounted semiannually at a 5 percent nominal annual rate of interest?

(A) $8,891

(B) $8,895

(C) $8,906

(D) $8,923

A

The answer is (D).

HP-10BII keystrokes: , C ALL; 12000, FV; 5, ÷, 2, =, I/YR; 6, x, 2, =, N; PV. Answer displayed: –8,922.67, becomes –8,923 when rounded to the nearest dollar.

(A), (B), and (C) are incorrect calculations.

How well did you know this?
1
Not at all
2
3
4
5
Perfectly
15
Q

What is the present value of the following series of payments based on a 5 percent compound annual interest rate? Timing of Payment Amount of Payment End of year 1 $1,000 End of year 2 3,000 End of year 3 2,000 End of year 4 0 End of year 5 2,000 End of year 6 2,000

(A) $6,968.20

(B) $8,460.63

(C) $9,044.28

(D) $10,000.00

A

The answer is (B).

HP-10BII keystrokes: , C ALL; 0, CFj; 1000, CFj; 3000, CFj; 2000, CFj; 0, CFj; 2000, CFj, 2, , Nj; 5, I/YR; , NPV. Answer displayed: 8,460.63.

(A), (C), and (D) are incorrect calculations.

How well did you know this?
1
Not at all
2
3
4
5
Perfectly
16
Q

At the end of year 5, what is the future value of the following series of deposits to a savings account that earns 6 percent compound annual interest? Timing of Deposit Amount of Deposit End of year 1 $1,000 End of year 2 1,000 End of year 3 0 End of year 4 2,000 End of year 5 2,000

(A) $4,912.10

(B) $5,096.82

(C) $5,206.82

(D) $6,573.49

A

The answer is (D).

HP-10BII Part 1 keystrokes: , C ALL; 0, CFj; 1000, CFj, 2, , Nj; 0, CFj; 2000, CFj, 2, , Nj; 6, I/YR; , NPV. Answer displayed: 4,912.10. Part 2 keystrokes: +/–, PV; 5, N; FV. Answer displayed: 6,573.49.

(A), (B), and (C) are incorrect calculations.

17
Q

Ashley’s current financial position statement indicates that on June 15, 2016 she had $50,000 of cash and cash equivalents and $2,000 of unpaid credit card balances. Assuming the total payments for her consumer, automobile, and mortgage loans will be $10,500 in 2016, what is Ashley’s liquidity ratio?

(A) 4.00

(B) 4.76

(C) 5.00

(D) 5.67

A

The answer is (A).

(A) is correct because the liquidity ratio includes credit card debt as well as all other short-term liabilities.

18
Q

Nancy is being offered a series of 6 annual payments in exchange for her ownership in a widget factory. The first payment of $200,000 will be payable immediately. Future payments will grow at a rate of 20 percent per year. If Nancy can earn 9 percent on her investments, what would the lump-sum equivalent of the offer be worth today?

(A) $868,715.27

(B) $956,383.78

(C) $1,546,699.49

(D) $1,702,788.43

A

The answer is (C).

HP-10BII keystrokes: , C ALL; , BEG/END (if BEGIN not displayed);200000, PMT; 1.09, ÷ 1.20, –, 1, x, 100, =, I/YR; 6, N; PV. Answer displayed: –1,546,699.49.

(A), (B), and (D) are incorrect calculations.

19
Q

What is the internal rate of return of a project with the following cash flows? Timing of Cash Flow Amount of Cash Flow Immediate $10,000 cash outflow End of year 1 5,000 cash outflow End of year 2 0 cash flow End of year 3 10,000 cash inflow End of year 4 20,000 cash inflow

(A) 17.61%

(B) 19.32%

(C) 21.54%

(D) 23.12%

A

The answer is (D).

HP-10BII keystrokes: , C ALL; 10000, +/–, CFj; 5000, +/–, CFj; 0, CFj; 10000, CFj; 20000, CFj; , IRR/YR. Answer displayed: 23.12.

(A), (B), and (C) are incorrect calculations.

20
Q

To what amount (rounded to the nearest dollar) will $8,000 grow in 3 years in a savings account that earns 8 percent interest compounded weekly?

(A) $10,078

(B) $10,123

(C) $10,146

(D) $10,168

A

The answer is (D).

HP-10BII keystrokes: , C ALL; 8000, +/–, PV; 8, ÷, 52, =, I/YR; 3, x, 52, = N; FV. Answer displayed: 10,168.12, becomes 10,168 when rounded to the nearest dollar.

(A), (B), and (C) are incorrect calculations.

21
Q

How much should an investor be willing to pay for a bond that pays $25 of interest every 6 months for the next 3 years and the $1,000 face amount at the end of 3 years if the investor requires a yield to maturity of 8 percent?

(A) $858.23

(B) $921.37

(C) $1,000.00

(D) $1,056.97

A

The answer is (B).

HP-10BII keystrokes: , C ALL; , BEG/END (if BEGIN displayed); 2, ,P/YR; 3, , xP/YR; 25, PMT; 1000, FV; 8, I/YR; PV. Answer displayed: –921.37. Then reset for one payment period per year: 1, , P/YR, C.

(A), (C), and (D) are incorrect calculations

22
Q

Dave has a credit card balance of $20,000, which carries an APR of 20%. He pays $400 per month towards retiring the balance. How long will it take Dave to reduce the balance to $0?

(A) 4.50 years

(B) 7.25 years

(C) 8.19 years

(D) 9.03 years

A

The answer is (D).

HP-10BII keystrokes: , C ALL; , BEG/END (if BEGIN displayed); 20000, PV; 20/12,=, I/YR; 400, PMT; N. Answer displayed: 108.399, divide by 12 (months). The answer is 9.03.

23
Q

Manny plans on saving $2,000 per year in a money market account that pays 2 percent yearly. He could be investing that same amount into a portfolio of mutual funds that averages 10 percent yearly. Over the course of 10 years, what will be the dollar difference between those two strategies?

(A) 8,754.31

(B) 9,975.41

(C) 10,875.51

(D) 11,423.61

A

The answer is (B).

HP-10BII keystrokes: , C ALL; 2000, +/–, PMT; 2, I/YR; 10, N; FV. Answer displayed: 21,899.49 for the 2 percent account. HP-10BII keystrokes: , C ALL; 2000, +/–, PMT; 10, I/YR; 10, N; FV. Answer displayed: 31,874.84 for the 10 percent account. Now 31,874.84- 21,899.49 =9,975.41